The people most likely to watch a televised debate between political candidates are the most committed members of the...

jingjingxiao11111@gmail.com on April 14, 2020

Why is D wrong? Thanks

Why is D wrong? Thanks

Reply
Create a free account to read and take part in forum discussions.

Already have an account? log in

shunhe on April 14, 2020

Hi @jingjingxiao11111@gmail.com,

Thanks for the question! Let’s first take a look at what the argument is telling us. Basically, we’re told that people who are most likely to watch a televised debate are already committed, and those who did watch who were uncommitted are generally undecided about who won. The argument then concludes that winning a televised debate doesn’t really do much for one’s chances of winning an election.

Now let’s take a look at what (D) tells us: the comments of participants in a televised debate might influence people’s voting behavior in unpredictable ways. The problem with (D) is that it rests on too may background assumptions, and when compared with (B), definitely isn’t the best answer here. Remember that on the LSAT, we can’t just accept an “ok” answer, we need to pick the best one out of the answer choices. Yes, if (D) were true, maybe candidates could be bolstered. But maybe there’d be no benefit as well. First off, we’re only told that their voting behavior “may” be influenced. Then, we’re told that these behavior changes are unpredictable, which is a step away from bolstering one’s chances at winning an election. (B), on the other hand, is a much clearer answer choice with a direct connection, since it touches on another group of people that the original argument failed to consider.

Hope this helps. Feel free to ask any further questions that you might have.